$sigma$-finite measure $mu$ so that $L^p(mu) subsetneq L^q(mu)$ (proper subset)











up vote
0
down vote

favorite












I'm looking for a $sigma$-finite measure $mu$ and a measure space so that for



$1 le p <q le infty$



$$L^p(mu) subsetneq L^q(mu)$$



I tried the following:



Let $1 le p <q le infty$ and $lambda$ the Lebesgue measure on $(1,infty)$ which is $sigma$-finite.



$x^alpha$ is integrable on $(1,infty) Leftrightarrow alpha <-1$.



Choose $b$ so that $1/q<b<1/p Leftrightarrow -bq<-1, -bp>-1$.



Then $x^{-b}chi_{(1,infty)}$$in L^q$ but $notin L^p$ because $x^{-bq}$ is integrable because the exponent $-bq<-1$ and $x^{-bp}$ isn't integrable because the exponent $-bp>-1$. Now I found a function that is in $L^p$ but not in $L^q$. But that doesn't really show that $L^p subsetneq L^q$, meaning $L^p$ is a proper subset of $L^q$, right (because I don't know if every element of $L^p$ is also an element of $L^q$)?



Thanks in advance!










share|cite|improve this question


























    up vote
    0
    down vote

    favorite












    I'm looking for a $sigma$-finite measure $mu$ and a measure space so that for



    $1 le p <q le infty$



    $$L^p(mu) subsetneq L^q(mu)$$



    I tried the following:



    Let $1 le p <q le infty$ and $lambda$ the Lebesgue measure on $(1,infty)$ which is $sigma$-finite.



    $x^alpha$ is integrable on $(1,infty) Leftrightarrow alpha <-1$.



    Choose $b$ so that $1/q<b<1/p Leftrightarrow -bq<-1, -bp>-1$.



    Then $x^{-b}chi_{(1,infty)}$$in L^q$ but $notin L^p$ because $x^{-bq}$ is integrable because the exponent $-bq<-1$ and $x^{-bp}$ isn't integrable because the exponent $-bp>-1$. Now I found a function that is in $L^p$ but not in $L^q$. But that doesn't really show that $L^p subsetneq L^q$, meaning $L^p$ is a proper subset of $L^q$, right (because I don't know if every element of $L^p$ is also an element of $L^q$)?



    Thanks in advance!










    share|cite|improve this question
























      up vote
      0
      down vote

      favorite









      up vote
      0
      down vote

      favorite











      I'm looking for a $sigma$-finite measure $mu$ and a measure space so that for



      $1 le p <q le infty$



      $$L^p(mu) subsetneq L^q(mu)$$



      I tried the following:



      Let $1 le p <q le infty$ and $lambda$ the Lebesgue measure on $(1,infty)$ which is $sigma$-finite.



      $x^alpha$ is integrable on $(1,infty) Leftrightarrow alpha <-1$.



      Choose $b$ so that $1/q<b<1/p Leftrightarrow -bq<-1, -bp>-1$.



      Then $x^{-b}chi_{(1,infty)}$$in L^q$ but $notin L^p$ because $x^{-bq}$ is integrable because the exponent $-bq<-1$ and $x^{-bp}$ isn't integrable because the exponent $-bp>-1$. Now I found a function that is in $L^p$ but not in $L^q$. But that doesn't really show that $L^p subsetneq L^q$, meaning $L^p$ is a proper subset of $L^q$, right (because I don't know if every element of $L^p$ is also an element of $L^q$)?



      Thanks in advance!










      share|cite|improve this question













      I'm looking for a $sigma$-finite measure $mu$ and a measure space so that for



      $1 le p <q le infty$



      $$L^p(mu) subsetneq L^q(mu)$$



      I tried the following:



      Let $1 le p <q le infty$ and $lambda$ the Lebesgue measure on $(1,infty)$ which is $sigma$-finite.



      $x^alpha$ is integrable on $(1,infty) Leftrightarrow alpha <-1$.



      Choose $b$ so that $1/q<b<1/p Leftrightarrow -bq<-1, -bp>-1$.



      Then $x^{-b}chi_{(1,infty)}$$in L^q$ but $notin L^p$ because $x^{-bq}$ is integrable because the exponent $-bq<-1$ and $x^{-bp}$ isn't integrable because the exponent $-bp>-1$. Now I found a function that is in $L^p$ but not in $L^q$. But that doesn't really show that $L^p subsetneq L^q$, meaning $L^p$ is a proper subset of $L^q$, right (because I don't know if every element of $L^p$ is also an element of $L^q$)?



      Thanks in advance!







      real-analysis analysis measure-theory






      share|cite|improve this question













      share|cite|improve this question











      share|cite|improve this question




      share|cite|improve this question










      asked Nov 13 at 17:04









      user610431

      627




      627






















          1 Answer
          1






          active

          oldest

          votes

















          up vote
          0
          down vote













          The inclusion $L^p(mu) subset L^q(mu)$ for $1leq p < q leq +infty$ holds if and only if
          $$
          inf{mu(A): Ainmathcal{M}, mu(A)>0} > 0.
          $$

          Hence your $mu$ must necessarily be an atomic measure.



          For example, you can consider the spaces $ell^p$. More precisely, you equip $mathbb{N}$ with the counting measure, so that
          $$
          ell^p := {x = (x_1, x_2, ldots): sum_{j=1}^infty |x_j|^p < infty}.
          $$






          share|cite|improve this answer























          • Thanks, but unfortunately I don't really understand what exactly you mean. Can you give me an example of such an measure?
            – user610431
            Nov 13 at 17:57










          • Done. (Edit in the main text.)
            – Rigel
            Nov 14 at 6:41











          Your Answer





          StackExchange.ifUsing("editor", function () {
          return StackExchange.using("mathjaxEditing", function () {
          StackExchange.MarkdownEditor.creationCallbacks.add(function (editor, postfix) {
          StackExchange.mathjaxEditing.prepareWmdForMathJax(editor, postfix, [["$", "$"], ["\\(","\\)"]]);
          });
          });
          }, "mathjax-editing");

          StackExchange.ready(function() {
          var channelOptions = {
          tags: "".split(" "),
          id: "69"
          };
          initTagRenderer("".split(" "), "".split(" "), channelOptions);

          StackExchange.using("externalEditor", function() {
          // Have to fire editor after snippets, if snippets enabled
          if (StackExchange.settings.snippets.snippetsEnabled) {
          StackExchange.using("snippets", function() {
          createEditor();
          });
          }
          else {
          createEditor();
          }
          });

          function createEditor() {
          StackExchange.prepareEditor({
          heartbeatType: 'answer',
          convertImagesToLinks: true,
          noModals: true,
          showLowRepImageUploadWarning: true,
          reputationToPostImages: 10,
          bindNavPrevention: true,
          postfix: "",
          imageUploader: {
          brandingHtml: "Powered by u003ca class="icon-imgur-white" href="https://imgur.com/"u003eu003c/au003e",
          contentPolicyHtml: "User contributions licensed under u003ca href="https://creativecommons.org/licenses/by-sa/3.0/"u003ecc by-sa 3.0 with attribution requiredu003c/au003e u003ca href="https://stackoverflow.com/legal/content-policy"u003e(content policy)u003c/au003e",
          allowUrls: true
          },
          noCode: true, onDemand: true,
          discardSelector: ".discard-answer"
          ,immediatelyShowMarkdownHelp:true
          });


          }
          });














           

          draft saved


          draft discarded


















          StackExchange.ready(
          function () {
          StackExchange.openid.initPostLogin('.new-post-login', 'https%3a%2f%2fmath.stackexchange.com%2fquestions%2f2996992%2fsigma-finite-measure-mu-so-that-lp-mu-subsetneq-lq-mu-proper-sub%23new-answer', 'question_page');
          }
          );

          Post as a guest















          Required, but never shown

























          1 Answer
          1






          active

          oldest

          votes








          1 Answer
          1






          active

          oldest

          votes









          active

          oldest

          votes






          active

          oldest

          votes








          up vote
          0
          down vote













          The inclusion $L^p(mu) subset L^q(mu)$ for $1leq p < q leq +infty$ holds if and only if
          $$
          inf{mu(A): Ainmathcal{M}, mu(A)>0} > 0.
          $$

          Hence your $mu$ must necessarily be an atomic measure.



          For example, you can consider the spaces $ell^p$. More precisely, you equip $mathbb{N}$ with the counting measure, so that
          $$
          ell^p := {x = (x_1, x_2, ldots): sum_{j=1}^infty |x_j|^p < infty}.
          $$






          share|cite|improve this answer























          • Thanks, but unfortunately I don't really understand what exactly you mean. Can you give me an example of such an measure?
            – user610431
            Nov 13 at 17:57










          • Done. (Edit in the main text.)
            – Rigel
            Nov 14 at 6:41















          up vote
          0
          down vote













          The inclusion $L^p(mu) subset L^q(mu)$ for $1leq p < q leq +infty$ holds if and only if
          $$
          inf{mu(A): Ainmathcal{M}, mu(A)>0} > 0.
          $$

          Hence your $mu$ must necessarily be an atomic measure.



          For example, you can consider the spaces $ell^p$. More precisely, you equip $mathbb{N}$ with the counting measure, so that
          $$
          ell^p := {x = (x_1, x_2, ldots): sum_{j=1}^infty |x_j|^p < infty}.
          $$






          share|cite|improve this answer























          • Thanks, but unfortunately I don't really understand what exactly you mean. Can you give me an example of such an measure?
            – user610431
            Nov 13 at 17:57










          • Done. (Edit in the main text.)
            – Rigel
            Nov 14 at 6:41













          up vote
          0
          down vote










          up vote
          0
          down vote









          The inclusion $L^p(mu) subset L^q(mu)$ for $1leq p < q leq +infty$ holds if and only if
          $$
          inf{mu(A): Ainmathcal{M}, mu(A)>0} > 0.
          $$

          Hence your $mu$ must necessarily be an atomic measure.



          For example, you can consider the spaces $ell^p$. More precisely, you equip $mathbb{N}$ with the counting measure, so that
          $$
          ell^p := {x = (x_1, x_2, ldots): sum_{j=1}^infty |x_j|^p < infty}.
          $$






          share|cite|improve this answer














          The inclusion $L^p(mu) subset L^q(mu)$ for $1leq p < q leq +infty$ holds if and only if
          $$
          inf{mu(A): Ainmathcal{M}, mu(A)>0} > 0.
          $$

          Hence your $mu$ must necessarily be an atomic measure.



          For example, you can consider the spaces $ell^p$. More precisely, you equip $mathbb{N}$ with the counting measure, so that
          $$
          ell^p := {x = (x_1, x_2, ldots): sum_{j=1}^infty |x_j|^p < infty}.
          $$







          share|cite|improve this answer














          share|cite|improve this answer



          share|cite|improve this answer








          edited Nov 14 at 6:40

























          answered Nov 13 at 17:30









          Rigel

          10.6k11319




          10.6k11319












          • Thanks, but unfortunately I don't really understand what exactly you mean. Can you give me an example of such an measure?
            – user610431
            Nov 13 at 17:57










          • Done. (Edit in the main text.)
            – Rigel
            Nov 14 at 6:41


















          • Thanks, but unfortunately I don't really understand what exactly you mean. Can you give me an example of such an measure?
            – user610431
            Nov 13 at 17:57










          • Done. (Edit in the main text.)
            – Rigel
            Nov 14 at 6:41
















          Thanks, but unfortunately I don't really understand what exactly you mean. Can you give me an example of such an measure?
          – user610431
          Nov 13 at 17:57




          Thanks, but unfortunately I don't really understand what exactly you mean. Can you give me an example of such an measure?
          – user610431
          Nov 13 at 17:57












          Done. (Edit in the main text.)
          – Rigel
          Nov 14 at 6:41




          Done. (Edit in the main text.)
          – Rigel
          Nov 14 at 6:41


















           

          draft saved


          draft discarded



















































           


          draft saved


          draft discarded














          StackExchange.ready(
          function () {
          StackExchange.openid.initPostLogin('.new-post-login', 'https%3a%2f%2fmath.stackexchange.com%2fquestions%2f2996992%2fsigma-finite-measure-mu-so-that-lp-mu-subsetneq-lq-mu-proper-sub%23new-answer', 'question_page');
          }
          );

          Post as a guest















          Required, but never shown





















































          Required, but never shown














          Required, but never shown












          Required, but never shown







          Required, but never shown

































          Required, but never shown














          Required, but never shown












          Required, but never shown







          Required, but never shown







          Popular posts from this blog

          How do I know what Microsoft account the skydrive app is syncing to?

          Grease: Live!

          When does type information flow backwards in C++?